Five Powers of Fives Produce Unique Pandigital Number…Solve for X..Tell me YCrack the Code #1Longest...

Why did nobody know who the Lord of this region was?

Is there any deeper thematic meaning to the white horse that Arya finds in The Bells (S08E05)?

SHAKE-128/256 or SHA3-256/512

Why does Taylor’s series “work”?

When did Britain learn about American independence?

How come Arya Stark didn't burn in Game of Thrones Season 8 Episode 5

Why did the soldiers of the North disobey Jon?

"Counterexample" for the Inverse function theorem

Why is Drogon so much better in battle than Rhaegal and Viserion?

Why does string strummed with finger sound different from the one strummed with pick?

Promotion comes with unexpected 24/7/365 on-call

Single word that parallels "Recent" when discussing the near future

Rushed passport - does my reason qualify?

FIFO data structure in pure C

What is the velocity distribution of the exhaust for a typical rocket engine?

What color to choose as "danger" if the main color of my app is red

Who is frowning in the sentence "Daisy looked at Tom frowning"?

Pedaling at different gear ratios on flat terrain: what's the point?

How can I make dummy text (like lipsum) grey?

He is the first man to arrive here

Non-African Click Languages

Why would company (decision makers) wait for someone to retire, rather than lay them off, when their role is no longer needed?

Is it possible to pass a pointer to an operator as an argument like a pointer to a function?

How can I fix the label locations on my tikzcd diagram?



Five Powers of Fives Produce Unique Pandigital Number…Solve for X..Tell me Y


Crack the Code #1Longest Calculator Word?A rather curious division machineVegas Street Magician Math TrickPassword CrackingFind a Strobogrammatic number, so if we square it, the result is a pandigit numberHoneydripping around the clockA mo-Roman samplerFind the equality with all digitsLong digital sequence. 16xxxxxxxxxxxxx61













2












$begingroup$


Given: Y is a Pan-digital Number (no zero, 1 to 9 only) ending in 3.



    Pan digital number consists of all 9 digits 1 to 9..each digit occurring only once as is the case here. Last digit is given as 3 and all other digits 1,2,4,5,6,7,8,9 can be anywhere in the number.


No googling, no computers, you don’t even need the calculator till the last step to calculate Y from X.



$Y = (X-1) ^ 5 + ( X + 7 ) ^ 5 + ( 2X + 6 ) ^ 5 + ( 4X + 3 ) ^ 5 + ( 5 X + 8) ^ 5$



Most concise, logical answer will be accepted.










share|improve this question











$endgroup$








  • 1




    $begingroup$
    In the future, for these kinds of alphametic/mathematical puzzles, I strongly encourage you to use MathJax to make the formatting look good, as it does now. Here's a meta post from Mathematics SE that gives a quick tutorial. I also recommend reading the Markdown help page for other formatting matters. Keep up the good work! :)
    $endgroup$
    – PiIsNot3
    58 mins ago










  • $begingroup$
    You may add the definition of pan-digital number (or put a link will do), I just know that term today
    $endgroup$
    – athin
    57 mins ago










  • $begingroup$
    Thx..will do in the future..didn’t have time to learn it fully
    $endgroup$
    – Uvc
    56 mins ago
















2












$begingroup$


Given: Y is a Pan-digital Number (no zero, 1 to 9 only) ending in 3.



    Pan digital number consists of all 9 digits 1 to 9..each digit occurring only once as is the case here. Last digit is given as 3 and all other digits 1,2,4,5,6,7,8,9 can be anywhere in the number.


No googling, no computers, you don’t even need the calculator till the last step to calculate Y from X.



$Y = (X-1) ^ 5 + ( X + 7 ) ^ 5 + ( 2X + 6 ) ^ 5 + ( 4X + 3 ) ^ 5 + ( 5 X + 8) ^ 5$



Most concise, logical answer will be accepted.










share|improve this question











$endgroup$








  • 1




    $begingroup$
    In the future, for these kinds of alphametic/mathematical puzzles, I strongly encourage you to use MathJax to make the formatting look good, as it does now. Here's a meta post from Mathematics SE that gives a quick tutorial. I also recommend reading the Markdown help page for other formatting matters. Keep up the good work! :)
    $endgroup$
    – PiIsNot3
    58 mins ago










  • $begingroup$
    You may add the definition of pan-digital number (or put a link will do), I just know that term today
    $endgroup$
    – athin
    57 mins ago










  • $begingroup$
    Thx..will do in the future..didn’t have time to learn it fully
    $endgroup$
    – Uvc
    56 mins ago














2












2








2





$begingroup$


Given: Y is a Pan-digital Number (no zero, 1 to 9 only) ending in 3.



    Pan digital number consists of all 9 digits 1 to 9..each digit occurring only once as is the case here. Last digit is given as 3 and all other digits 1,2,4,5,6,7,8,9 can be anywhere in the number.


No googling, no computers, you don’t even need the calculator till the last step to calculate Y from X.



$Y = (X-1) ^ 5 + ( X + 7 ) ^ 5 + ( 2X + 6 ) ^ 5 + ( 4X + 3 ) ^ 5 + ( 5 X + 8) ^ 5$



Most concise, logical answer will be accepted.










share|improve this question











$endgroup$




Given: Y is a Pan-digital Number (no zero, 1 to 9 only) ending in 3.



    Pan digital number consists of all 9 digits 1 to 9..each digit occurring only once as is the case here. Last digit is given as 3 and all other digits 1,2,4,5,6,7,8,9 can be anywhere in the number.


No googling, no computers, you don’t even need the calculator till the last step to calculate Y from X.



$Y = (X-1) ^ 5 + ( X + 7 ) ^ 5 + ( 2X + 6 ) ^ 5 + ( 4X + 3 ) ^ 5 + ( 5 X + 8) ^ 5$



Most concise, logical answer will be accepted.







mathematics no-computers






share|improve this question















share|improve this question













share|improve this question




share|improve this question








edited 51 mins ago







Uvc

















asked 1 hour ago









UvcUvc

59310




59310








  • 1




    $begingroup$
    In the future, for these kinds of alphametic/mathematical puzzles, I strongly encourage you to use MathJax to make the formatting look good, as it does now. Here's a meta post from Mathematics SE that gives a quick tutorial. I also recommend reading the Markdown help page for other formatting matters. Keep up the good work! :)
    $endgroup$
    – PiIsNot3
    58 mins ago










  • $begingroup$
    You may add the definition of pan-digital number (or put a link will do), I just know that term today
    $endgroup$
    – athin
    57 mins ago










  • $begingroup$
    Thx..will do in the future..didn’t have time to learn it fully
    $endgroup$
    – Uvc
    56 mins ago














  • 1




    $begingroup$
    In the future, for these kinds of alphametic/mathematical puzzles, I strongly encourage you to use MathJax to make the formatting look good, as it does now. Here's a meta post from Mathematics SE that gives a quick tutorial. I also recommend reading the Markdown help page for other formatting matters. Keep up the good work! :)
    $endgroup$
    – PiIsNot3
    58 mins ago










  • $begingroup$
    You may add the definition of pan-digital number (or put a link will do), I just know that term today
    $endgroup$
    – athin
    57 mins ago










  • $begingroup$
    Thx..will do in the future..didn’t have time to learn it fully
    $endgroup$
    – Uvc
    56 mins ago








1




1




$begingroup$
In the future, for these kinds of alphametic/mathematical puzzles, I strongly encourage you to use MathJax to make the formatting look good, as it does now. Here's a meta post from Mathematics SE that gives a quick tutorial. I also recommend reading the Markdown help page for other formatting matters. Keep up the good work! :)
$endgroup$
– PiIsNot3
58 mins ago




$begingroup$
In the future, for these kinds of alphametic/mathematical puzzles, I strongly encourage you to use MathJax to make the formatting look good, as it does now. Here's a meta post from Mathematics SE that gives a quick tutorial. I also recommend reading the Markdown help page for other formatting matters. Keep up the good work! :)
$endgroup$
– PiIsNot3
58 mins ago












$begingroup$
You may add the definition of pan-digital number (or put a link will do), I just know that term today
$endgroup$
– athin
57 mins ago




$begingroup$
You may add the definition of pan-digital number (or put a link will do), I just know that term today
$endgroup$
– athin
57 mins ago












$begingroup$
Thx..will do in the future..didn’t have time to learn it fully
$endgroup$
– Uvc
56 mins ago




$begingroup$
Thx..will do in the future..didn’t have time to learn it fully
$endgroup$
– Uvc
56 mins ago










3 Answers
3






active

oldest

votes


















1












$begingroup$

OK, now i realise its beauty...




for all $i$ from $0$ to $9$, $i^5$ mod $10$ = $i$




so to simply get the ending digit of $X$, the equation can be simplified:




$Y = (X-1) + ( X + 7 ) + ( 2X + 6 ) + ( 4X + 3 ) + ( 5 X + 8) $
$Y=13X +23$




More mod10-ing:




$Y=3X+3$




Sub $Y mod10 = 3$:




$3=3X+3$




Deducing $X mod10$:




$3X=0$
$X=0 (mod 10)$




Then, trial and error time, start from $X = $




$10$




Result (using a calculator in this very last step)




$Y=816725493$ BRAVO!!!







share|improve this answer











$endgroup$













  • $begingroup$
    ...yes.........
    $endgroup$
    – Uvc
    30 mins ago










  • $begingroup$
    beautiful question!!! @Uvc +1ed
    $endgroup$
    – Omega Krypton
    30 mins ago










  • $begingroup$
    Technically, you don't even need trial and error, as rot13(trareny zntavghqr pnyphyngvbaf zrna vg pbhyq bayl or 10. 20 znxrf bar grez 108^5, juvpu vf zber gura 9 qvtvgf).
    $endgroup$
    – Aranlyde
    13 mins ago



















1












$begingroup$

Without computers or calculators (at least until the very end), the answer is




$ X = boxed{10} $




The key here is to realize that




the units digit of $ Y $ being 3 limits our possibilities for $ X $ by a lot. Finding the last digit of a positive integer is the same as taking the integer modulo 10, so we will take the given expression for $ Y $ modulo 10, set it equal to 3, and solve for $ X. $




To do this, we




apply Euler's Theorem, which states that for all coprime integers $ a, n $ we have $ a^{phi(n)} equiv 1 ! pmod{n}, $ where $ phi(n) $ is Euler's totient function. For this problem, we'll rely on a similar equation $ a^{phi(n) + 1} equiv a ! pmod{n}, $ which works for any integers $ a, n, $ not just coprime.




Applying this theorem:




We have $ n = 10, $ so $$ a^{phi(10) + 1} equiv a^5 equiv a ! ! ! pmod{10}. $$ Thus, $$ begin{gather*} (X - 1)^5 + (X + 7)^5 + (2X + 6)^5 + (4X + 3)^5 + (5X + 8)^5 equiv 3 ! ! ! pmod{10} \ (X - 1) + (X + 7) + (2X + 6) + (4X + 3) + (5X + 8) equiv 3 ! ! ! pmod{10} \ 13X + 23 equiv 3 ! ! ! pmod{10} \ 3X equiv 0 ! ! ! pmod{10} \ X equiv 0 ! ! ! pmod{10} end{gather*} $$




Final answer:




We know now that $ X equiv 0 ! pmod{10} $ i.e. $ X $ is a multiple of 10 (0, 10, 20, 30, ...). Note that $ Y $ has exactly 9 digits, so $ X = 0$ can be ruled out since the sum will be less than 5 orders of magnitude. $ X = 10, $ however, does have the potential to come close, and by using a calculator we find that indeed it does work. Any higher values of $ X $ would cause it to have more than 9 digits, so this is our final and only answer.




For the record, the final solution for $ Y $ is




$ 816725493 = 9^5 + 17^5 + 26^5 + 43^5 + 58^5 $







share|improve this answer











$endgroup$













  • $begingroup$
    sorry, ninja-ed you, have an upvote!
    $endgroup$
    – Omega Krypton
    28 mins ago










  • $begingroup$
    @OmegaKrypton Well you did get a five minute head start.. I was still typing out my MathJax when your answer popped up! No matters, the OP will decide who gets the check mark :)
    $endgroup$
    – PiIsNot3
    22 mins ago





















1












$begingroup$

So this puzzle hinges on the fact that




$X^5mod10 = X$. ($1^5=1$, $2^5=32$, $3^5=243$, and so on.)




This means that




$(X+9)mod10+(X+7)mod10+(2X+6)mod10+(4X+3)mod10+(5X+8)mod10 = 3mod 10$ (as $(X-1)mod10 = (X+9)mod10$).




This simplifies to




$(13X+33)mod10=3$. Because of how the 3-times table works, this only works if $Xmod10=0$.




Looking only at




number of digits (for order-of-magnitude calculation), $8^5$ (for $X=0$), doesn't work, but $58^5$ (for $X=10$) does, thus making it the only possible solution.




The number is therefore




$816725943$.







share|improve this answer











$endgroup$













  • $begingroup$
    sorry, ninja-ed you, have an upvote!
    $endgroup$
    – Omega Krypton
    28 mins ago












Your Answer








StackExchange.ready(function() {
var channelOptions = {
tags: "".split(" "),
id: "559"
};
initTagRenderer("".split(" "), "".split(" "), channelOptions);

StackExchange.using("externalEditor", function() {
// Have to fire editor after snippets, if snippets enabled
if (StackExchange.settings.snippets.snippetsEnabled) {
StackExchange.using("snippets", function() {
createEditor();
});
}
else {
createEditor();
}
});

function createEditor() {
StackExchange.prepareEditor({
heartbeatType: 'answer',
autoActivateHeartbeat: false,
convertImagesToLinks: false,
noModals: true,
showLowRepImageUploadWarning: true,
reputationToPostImages: null,
bindNavPrevention: true,
postfix: "",
imageUploader: {
brandingHtml: "Powered by u003ca class="icon-imgur-white" href="https://imgur.com/"u003eu003c/au003e",
contentPolicyHtml: "User contributions licensed under u003ca href="https://creativecommons.org/licenses/by-sa/3.0/"u003ecc by-sa 3.0 with attribution requiredu003c/au003e u003ca href="https://stackoverflow.com/legal/content-policy"u003e(content policy)u003c/au003e",
allowUrls: true
},
noCode: true, onDemand: true,
discardSelector: ".discard-answer"
,immediatelyShowMarkdownHelp:true
});


}
});














draft saved

draft discarded


















StackExchange.ready(
function () {
StackExchange.openid.initPostLogin('.new-post-login', 'https%3a%2f%2fpuzzling.stackexchange.com%2fquestions%2f84005%2ffive-powers-of-fives-produce-unique-pandigital-number-solve-for-x-tell-me-y%23new-answer', 'question_page');
}
);

Post as a guest















Required, but never shown

























3 Answers
3






active

oldest

votes








3 Answers
3






active

oldest

votes









active

oldest

votes






active

oldest

votes









1












$begingroup$

OK, now i realise its beauty...




for all $i$ from $0$ to $9$, $i^5$ mod $10$ = $i$




so to simply get the ending digit of $X$, the equation can be simplified:




$Y = (X-1) + ( X + 7 ) + ( 2X + 6 ) + ( 4X + 3 ) + ( 5 X + 8) $
$Y=13X +23$




More mod10-ing:




$Y=3X+3$




Sub $Y mod10 = 3$:




$3=3X+3$




Deducing $X mod10$:




$3X=0$
$X=0 (mod 10)$




Then, trial and error time, start from $X = $




$10$




Result (using a calculator in this very last step)




$Y=816725493$ BRAVO!!!







share|improve this answer











$endgroup$













  • $begingroup$
    ...yes.........
    $endgroup$
    – Uvc
    30 mins ago










  • $begingroup$
    beautiful question!!! @Uvc +1ed
    $endgroup$
    – Omega Krypton
    30 mins ago










  • $begingroup$
    Technically, you don't even need trial and error, as rot13(trareny zntavghqr pnyphyngvbaf zrna vg pbhyq bayl or 10. 20 znxrf bar grez 108^5, juvpu vf zber gura 9 qvtvgf).
    $endgroup$
    – Aranlyde
    13 mins ago
















1












$begingroup$

OK, now i realise its beauty...




for all $i$ from $0$ to $9$, $i^5$ mod $10$ = $i$




so to simply get the ending digit of $X$, the equation can be simplified:




$Y = (X-1) + ( X + 7 ) + ( 2X + 6 ) + ( 4X + 3 ) + ( 5 X + 8) $
$Y=13X +23$




More mod10-ing:




$Y=3X+3$




Sub $Y mod10 = 3$:




$3=3X+3$




Deducing $X mod10$:




$3X=0$
$X=0 (mod 10)$




Then, trial and error time, start from $X = $




$10$




Result (using a calculator in this very last step)




$Y=816725493$ BRAVO!!!







share|improve this answer











$endgroup$













  • $begingroup$
    ...yes.........
    $endgroup$
    – Uvc
    30 mins ago










  • $begingroup$
    beautiful question!!! @Uvc +1ed
    $endgroup$
    – Omega Krypton
    30 mins ago










  • $begingroup$
    Technically, you don't even need trial and error, as rot13(trareny zntavghqr pnyphyngvbaf zrna vg pbhyq bayl or 10. 20 znxrf bar grez 108^5, juvpu vf zber gura 9 qvtvgf).
    $endgroup$
    – Aranlyde
    13 mins ago














1












1








1





$begingroup$

OK, now i realise its beauty...




for all $i$ from $0$ to $9$, $i^5$ mod $10$ = $i$




so to simply get the ending digit of $X$, the equation can be simplified:




$Y = (X-1) + ( X + 7 ) + ( 2X + 6 ) + ( 4X + 3 ) + ( 5 X + 8) $
$Y=13X +23$




More mod10-ing:




$Y=3X+3$




Sub $Y mod10 = 3$:




$3=3X+3$




Deducing $X mod10$:




$3X=0$
$X=0 (mod 10)$




Then, trial and error time, start from $X = $




$10$




Result (using a calculator in this very last step)




$Y=816725493$ BRAVO!!!







share|improve this answer











$endgroup$



OK, now i realise its beauty...




for all $i$ from $0$ to $9$, $i^5$ mod $10$ = $i$




so to simply get the ending digit of $X$, the equation can be simplified:




$Y = (X-1) + ( X + 7 ) + ( 2X + 6 ) + ( 4X + 3 ) + ( 5 X + 8) $
$Y=13X +23$




More mod10-ing:




$Y=3X+3$




Sub $Y mod10 = 3$:




$3=3X+3$




Deducing $X mod10$:




$3X=0$
$X=0 (mod 10)$




Then, trial and error time, start from $X = $




$10$




Result (using a calculator in this very last step)




$Y=816725493$ BRAVO!!!








share|improve this answer














share|improve this answer



share|improve this answer








edited 21 mins ago

























answered 37 mins ago









Omega KryptonOmega Krypton

6,1382953




6,1382953












  • $begingroup$
    ...yes.........
    $endgroup$
    – Uvc
    30 mins ago










  • $begingroup$
    beautiful question!!! @Uvc +1ed
    $endgroup$
    – Omega Krypton
    30 mins ago










  • $begingroup$
    Technically, you don't even need trial and error, as rot13(trareny zntavghqr pnyphyngvbaf zrna vg pbhyq bayl or 10. 20 znxrf bar grez 108^5, juvpu vf zber gura 9 qvtvgf).
    $endgroup$
    – Aranlyde
    13 mins ago


















  • $begingroup$
    ...yes.........
    $endgroup$
    – Uvc
    30 mins ago










  • $begingroup$
    beautiful question!!! @Uvc +1ed
    $endgroup$
    – Omega Krypton
    30 mins ago










  • $begingroup$
    Technically, you don't even need trial and error, as rot13(trareny zntavghqr pnyphyngvbaf zrna vg pbhyq bayl or 10. 20 znxrf bar grez 108^5, juvpu vf zber gura 9 qvtvgf).
    $endgroup$
    – Aranlyde
    13 mins ago
















$begingroup$
...yes.........
$endgroup$
– Uvc
30 mins ago




$begingroup$
...yes.........
$endgroup$
– Uvc
30 mins ago












$begingroup$
beautiful question!!! @Uvc +1ed
$endgroup$
– Omega Krypton
30 mins ago




$begingroup$
beautiful question!!! @Uvc +1ed
$endgroup$
– Omega Krypton
30 mins ago












$begingroup$
Technically, you don't even need trial and error, as rot13(trareny zntavghqr pnyphyngvbaf zrna vg pbhyq bayl or 10. 20 znxrf bar grez 108^5, juvpu vf zber gura 9 qvtvgf).
$endgroup$
– Aranlyde
13 mins ago




$begingroup$
Technically, you don't even need trial and error, as rot13(trareny zntavghqr pnyphyngvbaf zrna vg pbhyq bayl or 10. 20 znxrf bar grez 108^5, juvpu vf zber gura 9 qvtvgf).
$endgroup$
– Aranlyde
13 mins ago











1












$begingroup$

Without computers or calculators (at least until the very end), the answer is




$ X = boxed{10} $




The key here is to realize that




the units digit of $ Y $ being 3 limits our possibilities for $ X $ by a lot. Finding the last digit of a positive integer is the same as taking the integer modulo 10, so we will take the given expression for $ Y $ modulo 10, set it equal to 3, and solve for $ X. $




To do this, we




apply Euler's Theorem, which states that for all coprime integers $ a, n $ we have $ a^{phi(n)} equiv 1 ! pmod{n}, $ where $ phi(n) $ is Euler's totient function. For this problem, we'll rely on a similar equation $ a^{phi(n) + 1} equiv a ! pmod{n}, $ which works for any integers $ a, n, $ not just coprime.




Applying this theorem:




We have $ n = 10, $ so $$ a^{phi(10) + 1} equiv a^5 equiv a ! ! ! pmod{10}. $$ Thus, $$ begin{gather*} (X - 1)^5 + (X + 7)^5 + (2X + 6)^5 + (4X + 3)^5 + (5X + 8)^5 equiv 3 ! ! ! pmod{10} \ (X - 1) + (X + 7) + (2X + 6) + (4X + 3) + (5X + 8) equiv 3 ! ! ! pmod{10} \ 13X + 23 equiv 3 ! ! ! pmod{10} \ 3X equiv 0 ! ! ! pmod{10} \ X equiv 0 ! ! ! pmod{10} end{gather*} $$




Final answer:




We know now that $ X equiv 0 ! pmod{10} $ i.e. $ X $ is a multiple of 10 (0, 10, 20, 30, ...). Note that $ Y $ has exactly 9 digits, so $ X = 0$ can be ruled out since the sum will be less than 5 orders of magnitude. $ X = 10, $ however, does have the potential to come close, and by using a calculator we find that indeed it does work. Any higher values of $ X $ would cause it to have more than 9 digits, so this is our final and only answer.




For the record, the final solution for $ Y $ is




$ 816725493 = 9^5 + 17^5 + 26^5 + 43^5 + 58^5 $







share|improve this answer











$endgroup$













  • $begingroup$
    sorry, ninja-ed you, have an upvote!
    $endgroup$
    – Omega Krypton
    28 mins ago










  • $begingroup$
    @OmegaKrypton Well you did get a five minute head start.. I was still typing out my MathJax when your answer popped up! No matters, the OP will decide who gets the check mark :)
    $endgroup$
    – PiIsNot3
    22 mins ago


















1












$begingroup$

Without computers or calculators (at least until the very end), the answer is




$ X = boxed{10} $




The key here is to realize that




the units digit of $ Y $ being 3 limits our possibilities for $ X $ by a lot. Finding the last digit of a positive integer is the same as taking the integer modulo 10, so we will take the given expression for $ Y $ modulo 10, set it equal to 3, and solve for $ X. $




To do this, we




apply Euler's Theorem, which states that for all coprime integers $ a, n $ we have $ a^{phi(n)} equiv 1 ! pmod{n}, $ where $ phi(n) $ is Euler's totient function. For this problem, we'll rely on a similar equation $ a^{phi(n) + 1} equiv a ! pmod{n}, $ which works for any integers $ a, n, $ not just coprime.




Applying this theorem:




We have $ n = 10, $ so $$ a^{phi(10) + 1} equiv a^5 equiv a ! ! ! pmod{10}. $$ Thus, $$ begin{gather*} (X - 1)^5 + (X + 7)^5 + (2X + 6)^5 + (4X + 3)^5 + (5X + 8)^5 equiv 3 ! ! ! pmod{10} \ (X - 1) + (X + 7) + (2X + 6) + (4X + 3) + (5X + 8) equiv 3 ! ! ! pmod{10} \ 13X + 23 equiv 3 ! ! ! pmod{10} \ 3X equiv 0 ! ! ! pmod{10} \ X equiv 0 ! ! ! pmod{10} end{gather*} $$




Final answer:




We know now that $ X equiv 0 ! pmod{10} $ i.e. $ X $ is a multiple of 10 (0, 10, 20, 30, ...). Note that $ Y $ has exactly 9 digits, so $ X = 0$ can be ruled out since the sum will be less than 5 orders of magnitude. $ X = 10, $ however, does have the potential to come close, and by using a calculator we find that indeed it does work. Any higher values of $ X $ would cause it to have more than 9 digits, so this is our final and only answer.




For the record, the final solution for $ Y $ is




$ 816725493 = 9^5 + 17^5 + 26^5 + 43^5 + 58^5 $







share|improve this answer











$endgroup$













  • $begingroup$
    sorry, ninja-ed you, have an upvote!
    $endgroup$
    – Omega Krypton
    28 mins ago










  • $begingroup$
    @OmegaKrypton Well you did get a five minute head start.. I was still typing out my MathJax when your answer popped up! No matters, the OP will decide who gets the check mark :)
    $endgroup$
    – PiIsNot3
    22 mins ago
















1












1








1





$begingroup$

Without computers or calculators (at least until the very end), the answer is




$ X = boxed{10} $




The key here is to realize that




the units digit of $ Y $ being 3 limits our possibilities for $ X $ by a lot. Finding the last digit of a positive integer is the same as taking the integer modulo 10, so we will take the given expression for $ Y $ modulo 10, set it equal to 3, and solve for $ X. $




To do this, we




apply Euler's Theorem, which states that for all coprime integers $ a, n $ we have $ a^{phi(n)} equiv 1 ! pmod{n}, $ where $ phi(n) $ is Euler's totient function. For this problem, we'll rely on a similar equation $ a^{phi(n) + 1} equiv a ! pmod{n}, $ which works for any integers $ a, n, $ not just coprime.




Applying this theorem:




We have $ n = 10, $ so $$ a^{phi(10) + 1} equiv a^5 equiv a ! ! ! pmod{10}. $$ Thus, $$ begin{gather*} (X - 1)^5 + (X + 7)^5 + (2X + 6)^5 + (4X + 3)^5 + (5X + 8)^5 equiv 3 ! ! ! pmod{10} \ (X - 1) + (X + 7) + (2X + 6) + (4X + 3) + (5X + 8) equiv 3 ! ! ! pmod{10} \ 13X + 23 equiv 3 ! ! ! pmod{10} \ 3X equiv 0 ! ! ! pmod{10} \ X equiv 0 ! ! ! pmod{10} end{gather*} $$




Final answer:




We know now that $ X equiv 0 ! pmod{10} $ i.e. $ X $ is a multiple of 10 (0, 10, 20, 30, ...). Note that $ Y $ has exactly 9 digits, so $ X = 0$ can be ruled out since the sum will be less than 5 orders of magnitude. $ X = 10, $ however, does have the potential to come close, and by using a calculator we find that indeed it does work. Any higher values of $ X $ would cause it to have more than 9 digits, so this is our final and only answer.




For the record, the final solution for $ Y $ is




$ 816725493 = 9^5 + 17^5 + 26^5 + 43^5 + 58^5 $







share|improve this answer











$endgroup$



Without computers or calculators (at least until the very end), the answer is




$ X = boxed{10} $




The key here is to realize that




the units digit of $ Y $ being 3 limits our possibilities for $ X $ by a lot. Finding the last digit of a positive integer is the same as taking the integer modulo 10, so we will take the given expression for $ Y $ modulo 10, set it equal to 3, and solve for $ X. $




To do this, we




apply Euler's Theorem, which states that for all coprime integers $ a, n $ we have $ a^{phi(n)} equiv 1 ! pmod{n}, $ where $ phi(n) $ is Euler's totient function. For this problem, we'll rely on a similar equation $ a^{phi(n) + 1} equiv a ! pmod{n}, $ which works for any integers $ a, n, $ not just coprime.




Applying this theorem:




We have $ n = 10, $ so $$ a^{phi(10) + 1} equiv a^5 equiv a ! ! ! pmod{10}. $$ Thus, $$ begin{gather*} (X - 1)^5 + (X + 7)^5 + (2X + 6)^5 + (4X + 3)^5 + (5X + 8)^5 equiv 3 ! ! ! pmod{10} \ (X - 1) + (X + 7) + (2X + 6) + (4X + 3) + (5X + 8) equiv 3 ! ! ! pmod{10} \ 13X + 23 equiv 3 ! ! ! pmod{10} \ 3X equiv 0 ! ! ! pmod{10} \ X equiv 0 ! ! ! pmod{10} end{gather*} $$




Final answer:




We know now that $ X equiv 0 ! pmod{10} $ i.e. $ X $ is a multiple of 10 (0, 10, 20, 30, ...). Note that $ Y $ has exactly 9 digits, so $ X = 0$ can be ruled out since the sum will be less than 5 orders of magnitude. $ X = 10, $ however, does have the potential to come close, and by using a calculator we find that indeed it does work. Any higher values of $ X $ would cause it to have more than 9 digits, so this is our final and only answer.




For the record, the final solution for $ Y $ is




$ 816725493 = 9^5 + 17^5 + 26^5 + 43^5 + 58^5 $








share|improve this answer














share|improve this answer



share|improve this answer








edited 20 mins ago

























answered 28 mins ago









PiIsNot3PiIsNot3

4,323952




4,323952












  • $begingroup$
    sorry, ninja-ed you, have an upvote!
    $endgroup$
    – Omega Krypton
    28 mins ago










  • $begingroup$
    @OmegaKrypton Well you did get a five minute head start.. I was still typing out my MathJax when your answer popped up! No matters, the OP will decide who gets the check mark :)
    $endgroup$
    – PiIsNot3
    22 mins ago




















  • $begingroup$
    sorry, ninja-ed you, have an upvote!
    $endgroup$
    – Omega Krypton
    28 mins ago










  • $begingroup$
    @OmegaKrypton Well you did get a five minute head start.. I was still typing out my MathJax when your answer popped up! No matters, the OP will decide who gets the check mark :)
    $endgroup$
    – PiIsNot3
    22 mins ago


















$begingroup$
sorry, ninja-ed you, have an upvote!
$endgroup$
– Omega Krypton
28 mins ago




$begingroup$
sorry, ninja-ed you, have an upvote!
$endgroup$
– Omega Krypton
28 mins ago












$begingroup$
@OmegaKrypton Well you did get a five minute head start.. I was still typing out my MathJax when your answer popped up! No matters, the OP will decide who gets the check mark :)
$endgroup$
– PiIsNot3
22 mins ago






$begingroup$
@OmegaKrypton Well you did get a five minute head start.. I was still typing out my MathJax when your answer popped up! No matters, the OP will decide who gets the check mark :)
$endgroup$
– PiIsNot3
22 mins ago













1












$begingroup$

So this puzzle hinges on the fact that




$X^5mod10 = X$. ($1^5=1$, $2^5=32$, $3^5=243$, and so on.)




This means that




$(X+9)mod10+(X+7)mod10+(2X+6)mod10+(4X+3)mod10+(5X+8)mod10 = 3mod 10$ (as $(X-1)mod10 = (X+9)mod10$).




This simplifies to




$(13X+33)mod10=3$. Because of how the 3-times table works, this only works if $Xmod10=0$.




Looking only at




number of digits (for order-of-magnitude calculation), $8^5$ (for $X=0$), doesn't work, but $58^5$ (for $X=10$) does, thus making it the only possible solution.




The number is therefore




$816725943$.







share|improve this answer











$endgroup$













  • $begingroup$
    sorry, ninja-ed you, have an upvote!
    $endgroup$
    – Omega Krypton
    28 mins ago
















1












$begingroup$

So this puzzle hinges on the fact that




$X^5mod10 = X$. ($1^5=1$, $2^5=32$, $3^5=243$, and so on.)




This means that




$(X+9)mod10+(X+7)mod10+(2X+6)mod10+(4X+3)mod10+(5X+8)mod10 = 3mod 10$ (as $(X-1)mod10 = (X+9)mod10$).




This simplifies to




$(13X+33)mod10=3$. Because of how the 3-times table works, this only works if $Xmod10=0$.




Looking only at




number of digits (for order-of-magnitude calculation), $8^5$ (for $X=0$), doesn't work, but $58^5$ (for $X=10$) does, thus making it the only possible solution.




The number is therefore




$816725943$.







share|improve this answer











$endgroup$













  • $begingroup$
    sorry, ninja-ed you, have an upvote!
    $endgroup$
    – Omega Krypton
    28 mins ago














1












1








1





$begingroup$

So this puzzle hinges on the fact that




$X^5mod10 = X$. ($1^5=1$, $2^5=32$, $3^5=243$, and so on.)




This means that




$(X+9)mod10+(X+7)mod10+(2X+6)mod10+(4X+3)mod10+(5X+8)mod10 = 3mod 10$ (as $(X-1)mod10 = (X+9)mod10$).




This simplifies to




$(13X+33)mod10=3$. Because of how the 3-times table works, this only works if $Xmod10=0$.




Looking only at




number of digits (for order-of-magnitude calculation), $8^5$ (for $X=0$), doesn't work, but $58^5$ (for $X=10$) does, thus making it the only possible solution.




The number is therefore




$816725943$.







share|improve this answer











$endgroup$



So this puzzle hinges on the fact that




$X^5mod10 = X$. ($1^5=1$, $2^5=32$, $3^5=243$, and so on.)




This means that




$(X+9)mod10+(X+7)mod10+(2X+6)mod10+(4X+3)mod10+(5X+8)mod10 = 3mod 10$ (as $(X-1)mod10 = (X+9)mod10$).




This simplifies to




$(13X+33)mod10=3$. Because of how the 3-times table works, this only works if $Xmod10=0$.




Looking only at




number of digits (for order-of-magnitude calculation), $8^5$ (for $X=0$), doesn't work, but $58^5$ (for $X=10$) does, thus making it the only possible solution.




The number is therefore




$816725943$.








share|improve this answer














share|improve this answer



share|improve this answer








edited 10 mins ago

























answered 29 mins ago









AranlydeAranlyde

656212




656212












  • $begingroup$
    sorry, ninja-ed you, have an upvote!
    $endgroup$
    – Omega Krypton
    28 mins ago


















  • $begingroup$
    sorry, ninja-ed you, have an upvote!
    $endgroup$
    – Omega Krypton
    28 mins ago
















$begingroup$
sorry, ninja-ed you, have an upvote!
$endgroup$
– Omega Krypton
28 mins ago




$begingroup$
sorry, ninja-ed you, have an upvote!
$endgroup$
– Omega Krypton
28 mins ago


















draft saved

draft discarded




















































Thanks for contributing an answer to Puzzling Stack Exchange!


  • Please be sure to answer the question. Provide details and share your research!

But avoid



  • Asking for help, clarification, or responding to other answers.

  • Making statements based on opinion; back them up with references or personal experience.


Use MathJax to format equations. MathJax reference.


To learn more, see our tips on writing great answers.




draft saved


draft discarded














StackExchange.ready(
function () {
StackExchange.openid.initPostLogin('.new-post-login', 'https%3a%2f%2fpuzzling.stackexchange.com%2fquestions%2f84005%2ffive-powers-of-fives-produce-unique-pandigital-number-solve-for-x-tell-me-y%23new-answer', 'question_page');
}
);

Post as a guest















Required, but never shown





















































Required, but never shown














Required, but never shown












Required, but never shown







Required, but never shown

































Required, but never shown














Required, but never shown












Required, but never shown







Required, but never shown







Popular posts from this blog

Taj Mahal Inhaltsverzeichnis Aufbau | Geschichte | 350-Jahr-Feier | Heutige Bedeutung | Siehe auch |...

Baia Sprie Cuprins Etimologie | Istorie | Demografie | Politică și administrație | Arii naturale...

Nicolae Petrescu-Găină Cuprins Biografie | Opera | In memoriam | Varia | Controverse, incertitudini...